Masa efectiva en el sistema Spring-with-masa/masa

Supongamos que tienes una partícula de masa metro fijado a un resorte de masa metro 0 que, a su vez, está fijada a alguna pared. Estoy tratando de calcular la masa efectiva metro que aparece en la ley de movimiento de la partícula (supongamos que el sistema está aislado):

metro X ¨ = k ( X X 0 ) .
He leído en alguna parte que esto debería ser metro = metro + metro 0 / 3 , pero obtengo un resultado diferente.

Mi razonamiento es el siguiente. Supongamos que la partícula está en la posición X . La longitud del resorte es X y podemos suponer que su centro de masa está en X / 2 . Así que el centro de masa del resorte/partícula está en:

X = metro 0 2 + metro metro + metro 0 X
Derivando dos tiempos obtenemos
X ¨ = metro 0 2 + metro metro + metro 0 X ¨
Ahora, la única fuerza externa que causa la aceleración hacia el centro de masa es la reacción del techo a la fuerza elástica, eso es exactamente k ( X X 0 ) . De este modo:
k ( X X 0 ) = ( metro + metro 0 ) X ¨ = ( metro 0 2 + metro ) X ¨
y entonces obtengo:
metro = metro 0 2 + metro .

¿Podría señalar dónde me equivoco (si es que lo estoy) y posiblemente cómo se demuestra el resultado?

Se equivoca al suponer que puede ver el problema como una masa mejorada con un centro de masa desplazado. El resorte se comporta de manera muy diferente. La solución a esto es bastante larga: mire este sitio web para obtener una solución extensa: mathrec.org/old/2001dec/solutions.html La respuesta que recibió proviene de taylor expandiendo la solución a los eqs. de movimiento cuando m >> m0
Gracias por el enlace, me ayudó mucho. Pero todavía no entiendo dónde falla mi razonamiento. ¿Es solo una coincidencia que funcione para el caso estático, pero no para el dinámico? Quiero decir, en el caso estático parece que no hay error. También sugiero que publique esto como una respuesta para que pueda aceptarlo.
metro X ¨ = Está Mal. No se puede tomar un resorte masivo y una ley de fuerza lineal. Tienes que resolver una ecuación diferencial.
Me he preguntado lo mismo, y recuerdo haber resuelto este problema hace unos años. Pero cómo, no lo recuerdo. También recuerdo que la fracción 1/3 solo es válida para frecuencias bajas (< frec. nat.). A medida que aumentan las frecuencias, también aumenta la relación.
He escrito esto pero produce una proporción de 4 π 2 = 0.405 .
Tenga en cuenta que en el diseño de trenes de válvulas, los resortes helicoidales obtienen el 1 3 tratamiento, pero los manantiales de colmena obtienen una proporción más alta. En algunos casos hasta 1 2 .
@ja72: lo siento, no he podido responder hasta ahora, sin embargo, ese enlace que publicaste es demasiado avanzado para mí, ya que nunca había visto, por ejemplo, la ecuación de onda (bueno, hasta ayer). Me lo guardo para una futura lectura.
Energía cinética T = METRO 2 v 2 0 1 α 2 d α = METRO 6 v 2 de resorte fijo en un extremo.

Respuestas (1)

Puedes entender de una manera sencilla el factor 1 / 3 lo que le da la solución aproximada en el régimen de baja frecuencia (más sobre esto más adelante) de la siguiente manera. Comience escribiendo la energía cinética de su sistema como:

k = 1 2 metro d ˙ ( ) 2 + 0 1 2 λ d ˙ ( tu ) 2 d tu

dónde d ( tu ) es el desplazamiento de la punta del resorte que está en el X = tu posición en la configuración de equilibrio y λ = metro 0 / la densidad de masa lineal del resorte. El resorte tiene longitud cuando no está estirado.

Si supone un movimiento armónico de la masa a una frecuencia muy baja, el estiramiento del resorte será aproximadamente uniforme, lo que significa

d ( tu ) = tu d ( )

Aceptando esta aproximación sustituyendo en la expresión de la energía cinética se obtiene

k = 1 2 metro d ˙ ( ) 2 + 1 2 metro 0 d ˙ ( ) 2 0 tu 2 2 d tu

y después de una integración

k = 1 2 ( metro + 1 3 metro 0 ) d ˙ ( ) 2

cual es el resultado esperado.

El sistema tiene un número infinito de grados de libertad, lo que significa que tendrá un número infinito de modos de oscilación. Si metro metro 0 el modo de frecuencia más baja se describirá aproximadamente como una oscilación de la masa con un estiramiento uniforme del resorte. En los modos de frecuencia más alta, la masa será casi fija y habrá una onda elástica casi estacionaria en el resorte.

La falla en tu razonamiento consiste en suponer que la fuerza externa aplicada al sistema masa+resorte es k ( X X 0 ) . La fuerza aplicada es realmente la tensión del resorte en su punto fijo, que no es k ( X X 0 ) para un resorte con masa cuando hay aceleraciones.